A curious equality of integrals involving the prime counting function?How many primes does this sequence...

Existence of Riemann surface, holomorphic maps

Is using an 'empty' metaphor considered bad style?

Why does magnet wire need to be insulated?

Is there a defined priority for pattern matching?

Why is Agricola named as such?

Why was Lupin comfortable with saying Voldemort's name?

How does one write from a minority culture? A question on cultural references

Is "the fire consumed everything on its way" correct?

What is a DAG (Graph Theory)?

Is there a lava-breathing lizard creature (that could be worshipped by a cult) in 5e?

Do "fields" always combine by addition?

What is a good reason for every spaceship to carry a weapon on board?

How to politely refuse in-office gym instructor for steroids and protein

Why do we have to make "peinlich" start with a capital letter and also end with -s in this sentence?

What will happen if Parliament votes "no" on each of the Brexit-related votes to be held on the 12th, 13th and 14th of March?

Building an exterior wall within an exterior wall for insulation

How to not let the Identify spell spoil everything?

What makes papers publishable in top-tier journals?

Removing whitespace between consecutive numbers

Why are all my replica super soldiers young adults or old teenagers?

Can you tell from a blurry photo if focus was too close or too far?

How do I prevent a homebrew Grappling Hook feature from trivializing Tomb of Annihilation?

Has Britain negotiated with any other countries outside the EU in preparation for the exit?

Hilchos Shabbos English Sefer



A curious equality of integrals involving the prime counting function?


How many primes does this sequence find?Tight bounds on the prime counting functionDirichlet prime counting function?closed form for integrals involving error functiona practical prime counting functionPrime counting functionRestricted equality involving prime numbersPrime counting function formulasProof for a prime number formula involving the prime counting functionProperty of Prime Counting FunctionApproximating the prime counting function













9












$begingroup$


This post discusses the integral,
$$I(k)=int_0^kpi(x)pi(k-x)dx$$



where $pi(x)$ is the prime-counting function. For example,
$$I(13)=int_0^{13}pi(x)pi(13-x)dx = 73$$



Using WolframAlpha, the first 50 values for $k=1,2,3,dots$ are,



$$I(k) = 0, 0, 0, 0, 1, 4, 8, 14, 22, 32, 45, 58, 73, 90, 110, 132, 158, 184, 214, 246, 282, 320, 363, 406, 455, 506, 562, 618, 678, 738, 804, 872, 944, 1018, 1099, 1180, 1269, 1358, 1450, 1544, 1644, 1744, 1852, 1962, 2078, 2196, 2321, 2446, 2581, 2718,dots$$



While trying to find if the above sequence obeyed a pattern, I noticed a rather unexpected relationship:






Q: For all $n>0$, is it true,
$$I(6n+4) - 2,I(6n+5) + I(6n+6) overset{color{red}?}= 0$$




Example, for $n=1,2$, then
$$I(10)-2I(11)+I(12)=32-2*45+58 = 0$$
$$I(16)-2I(17)+I(18)=132-2*158+184= 0$$
and so on.










share|cite|improve this question











$endgroup$












  • $begingroup$
    Note your proposed equation doesn't hold for $n = 0$ as $I(4) = 0$, $I(5) = 1$ and $I(6) = 4$.
    $endgroup$
    – John Omielan
    1 hour ago










  • $begingroup$
    @JohnOmielan: A typo. I meant all $n>0$. I will correct it.
    $endgroup$
    – Tito Piezas III
    1 hour ago










  • $begingroup$
    I have checked to confirm what you're asking is true for $n$ up to $18$. However, I have my doubts it'll always work, partially because it doesn't work for $n = 0$. Also, a similar type condition is that $I(6n) - 2I(6n + 1) + I(6n + 2) = 2$, which holds for $1 le n le 5$, but at $n = 6$, the LHS becomes $0$ instead. If I get a chance, I will investigate your equation to see if I can figure out why it's true for at least the first $18$ values and, more importantly, will it always stay true. Regardless, though, it's an excellent observation you've made, even if it doesn't always hold.
    $endgroup$
    – John Omielan
    49 mins ago












  • $begingroup$
    I checked your result up to $n=533$ (for $n geq 534$, I have problems. Would you be interested by a huge table of $I(k)$ (I was able to generate it up to $k=540$). This is a very interesting problem.
    $endgroup$
    – Claude Leibovici
    30 mins ago












  • $begingroup$
    @ClaudeLeibovici: Thanks for checking, Claude! However, that table would be too huge for MSE. :)
    $endgroup$
    – Tito Piezas III
    27 mins ago
















9












$begingroup$


This post discusses the integral,
$$I(k)=int_0^kpi(x)pi(k-x)dx$$



where $pi(x)$ is the prime-counting function. For example,
$$I(13)=int_0^{13}pi(x)pi(13-x)dx = 73$$



Using WolframAlpha, the first 50 values for $k=1,2,3,dots$ are,



$$I(k) = 0, 0, 0, 0, 1, 4, 8, 14, 22, 32, 45, 58, 73, 90, 110, 132, 158, 184, 214, 246, 282, 320, 363, 406, 455, 506, 562, 618, 678, 738, 804, 872, 944, 1018, 1099, 1180, 1269, 1358, 1450, 1544, 1644, 1744, 1852, 1962, 2078, 2196, 2321, 2446, 2581, 2718,dots$$



While trying to find if the above sequence obeyed a pattern, I noticed a rather unexpected relationship:






Q: For all $n>0$, is it true,
$$I(6n+4) - 2,I(6n+5) + I(6n+6) overset{color{red}?}= 0$$




Example, for $n=1,2$, then
$$I(10)-2I(11)+I(12)=32-2*45+58 = 0$$
$$I(16)-2I(17)+I(18)=132-2*158+184= 0$$
and so on.










share|cite|improve this question











$endgroup$












  • $begingroup$
    Note your proposed equation doesn't hold for $n = 0$ as $I(4) = 0$, $I(5) = 1$ and $I(6) = 4$.
    $endgroup$
    – John Omielan
    1 hour ago










  • $begingroup$
    @JohnOmielan: A typo. I meant all $n>0$. I will correct it.
    $endgroup$
    – Tito Piezas III
    1 hour ago










  • $begingroup$
    I have checked to confirm what you're asking is true for $n$ up to $18$. However, I have my doubts it'll always work, partially because it doesn't work for $n = 0$. Also, a similar type condition is that $I(6n) - 2I(6n + 1) + I(6n + 2) = 2$, which holds for $1 le n le 5$, but at $n = 6$, the LHS becomes $0$ instead. If I get a chance, I will investigate your equation to see if I can figure out why it's true for at least the first $18$ values and, more importantly, will it always stay true. Regardless, though, it's an excellent observation you've made, even if it doesn't always hold.
    $endgroup$
    – John Omielan
    49 mins ago












  • $begingroup$
    I checked your result up to $n=533$ (for $n geq 534$, I have problems. Would you be interested by a huge table of $I(k)$ (I was able to generate it up to $k=540$). This is a very interesting problem.
    $endgroup$
    – Claude Leibovici
    30 mins ago












  • $begingroup$
    @ClaudeLeibovici: Thanks for checking, Claude! However, that table would be too huge for MSE. :)
    $endgroup$
    – Tito Piezas III
    27 mins ago














9












9








9


5



$begingroup$


This post discusses the integral,
$$I(k)=int_0^kpi(x)pi(k-x)dx$$



where $pi(x)$ is the prime-counting function. For example,
$$I(13)=int_0^{13}pi(x)pi(13-x)dx = 73$$



Using WolframAlpha, the first 50 values for $k=1,2,3,dots$ are,



$$I(k) = 0, 0, 0, 0, 1, 4, 8, 14, 22, 32, 45, 58, 73, 90, 110, 132, 158, 184, 214, 246, 282, 320, 363, 406, 455, 506, 562, 618, 678, 738, 804, 872, 944, 1018, 1099, 1180, 1269, 1358, 1450, 1544, 1644, 1744, 1852, 1962, 2078, 2196, 2321, 2446, 2581, 2718,dots$$



While trying to find if the above sequence obeyed a pattern, I noticed a rather unexpected relationship:






Q: For all $n>0$, is it true,
$$I(6n+4) - 2,I(6n+5) + I(6n+6) overset{color{red}?}= 0$$




Example, for $n=1,2$, then
$$I(10)-2I(11)+I(12)=32-2*45+58 = 0$$
$$I(16)-2I(17)+I(18)=132-2*158+184= 0$$
and so on.










share|cite|improve this question











$endgroup$




This post discusses the integral,
$$I(k)=int_0^kpi(x)pi(k-x)dx$$



where $pi(x)$ is the prime-counting function. For example,
$$I(13)=int_0^{13}pi(x)pi(13-x)dx = 73$$



Using WolframAlpha, the first 50 values for $k=1,2,3,dots$ are,



$$I(k) = 0, 0, 0, 0, 1, 4, 8, 14, 22, 32, 45, 58, 73, 90, 110, 132, 158, 184, 214, 246, 282, 320, 363, 406, 455, 506, 562, 618, 678, 738, 804, 872, 944, 1018, 1099, 1180, 1269, 1358, 1450, 1544, 1644, 1744, 1852, 1962, 2078, 2196, 2321, 2446, 2581, 2718,dots$$



While trying to find if the above sequence obeyed a pattern, I noticed a rather unexpected relationship:






Q: For all $n>0$, is it true,
$$I(6n+4) - 2,I(6n+5) + I(6n+6) overset{color{red}?}= 0$$




Example, for $n=1,2$, then
$$I(10)-2I(11)+I(12)=32-2*45+58 = 0$$
$$I(16)-2I(17)+I(18)=132-2*158+184= 0$$
and so on.







integration definite-integrals prime-numbers






share|cite|improve this question















share|cite|improve this question













share|cite|improve this question




share|cite|improve this question








edited 14 mins ago







Tito Piezas III

















asked 1 hour ago









Tito Piezas IIITito Piezas III

27.4k366174




27.4k366174












  • $begingroup$
    Note your proposed equation doesn't hold for $n = 0$ as $I(4) = 0$, $I(5) = 1$ and $I(6) = 4$.
    $endgroup$
    – John Omielan
    1 hour ago










  • $begingroup$
    @JohnOmielan: A typo. I meant all $n>0$. I will correct it.
    $endgroup$
    – Tito Piezas III
    1 hour ago










  • $begingroup$
    I have checked to confirm what you're asking is true for $n$ up to $18$. However, I have my doubts it'll always work, partially because it doesn't work for $n = 0$. Also, a similar type condition is that $I(6n) - 2I(6n + 1) + I(6n + 2) = 2$, which holds for $1 le n le 5$, but at $n = 6$, the LHS becomes $0$ instead. If I get a chance, I will investigate your equation to see if I can figure out why it's true for at least the first $18$ values and, more importantly, will it always stay true. Regardless, though, it's an excellent observation you've made, even if it doesn't always hold.
    $endgroup$
    – John Omielan
    49 mins ago












  • $begingroup$
    I checked your result up to $n=533$ (for $n geq 534$, I have problems. Would you be interested by a huge table of $I(k)$ (I was able to generate it up to $k=540$). This is a very interesting problem.
    $endgroup$
    – Claude Leibovici
    30 mins ago












  • $begingroup$
    @ClaudeLeibovici: Thanks for checking, Claude! However, that table would be too huge for MSE. :)
    $endgroup$
    – Tito Piezas III
    27 mins ago


















  • $begingroup$
    Note your proposed equation doesn't hold for $n = 0$ as $I(4) = 0$, $I(5) = 1$ and $I(6) = 4$.
    $endgroup$
    – John Omielan
    1 hour ago










  • $begingroup$
    @JohnOmielan: A typo. I meant all $n>0$. I will correct it.
    $endgroup$
    – Tito Piezas III
    1 hour ago










  • $begingroup$
    I have checked to confirm what you're asking is true for $n$ up to $18$. However, I have my doubts it'll always work, partially because it doesn't work for $n = 0$. Also, a similar type condition is that $I(6n) - 2I(6n + 1) + I(6n + 2) = 2$, which holds for $1 le n le 5$, but at $n = 6$, the LHS becomes $0$ instead. If I get a chance, I will investigate your equation to see if I can figure out why it's true for at least the first $18$ values and, more importantly, will it always stay true. Regardless, though, it's an excellent observation you've made, even if it doesn't always hold.
    $endgroup$
    – John Omielan
    49 mins ago












  • $begingroup$
    I checked your result up to $n=533$ (for $n geq 534$, I have problems. Would you be interested by a huge table of $I(k)$ (I was able to generate it up to $k=540$). This is a very interesting problem.
    $endgroup$
    – Claude Leibovici
    30 mins ago












  • $begingroup$
    @ClaudeLeibovici: Thanks for checking, Claude! However, that table would be too huge for MSE. :)
    $endgroup$
    – Tito Piezas III
    27 mins ago
















$begingroup$
Note your proposed equation doesn't hold for $n = 0$ as $I(4) = 0$, $I(5) = 1$ and $I(6) = 4$.
$endgroup$
– John Omielan
1 hour ago




$begingroup$
Note your proposed equation doesn't hold for $n = 0$ as $I(4) = 0$, $I(5) = 1$ and $I(6) = 4$.
$endgroup$
– John Omielan
1 hour ago












$begingroup$
@JohnOmielan: A typo. I meant all $n>0$. I will correct it.
$endgroup$
– Tito Piezas III
1 hour ago




$begingroup$
@JohnOmielan: A typo. I meant all $n>0$. I will correct it.
$endgroup$
– Tito Piezas III
1 hour ago












$begingroup$
I have checked to confirm what you're asking is true for $n$ up to $18$. However, I have my doubts it'll always work, partially because it doesn't work for $n = 0$. Also, a similar type condition is that $I(6n) - 2I(6n + 1) + I(6n + 2) = 2$, which holds for $1 le n le 5$, but at $n = 6$, the LHS becomes $0$ instead. If I get a chance, I will investigate your equation to see if I can figure out why it's true for at least the first $18$ values and, more importantly, will it always stay true. Regardless, though, it's an excellent observation you've made, even if it doesn't always hold.
$endgroup$
– John Omielan
49 mins ago






$begingroup$
I have checked to confirm what you're asking is true for $n$ up to $18$. However, I have my doubts it'll always work, partially because it doesn't work for $n = 0$. Also, a similar type condition is that $I(6n) - 2I(6n + 1) + I(6n + 2) = 2$, which holds for $1 le n le 5$, but at $n = 6$, the LHS becomes $0$ instead. If I get a chance, I will investigate your equation to see if I can figure out why it's true for at least the first $18$ values and, more importantly, will it always stay true. Regardless, though, it's an excellent observation you've made, even if it doesn't always hold.
$endgroup$
– John Omielan
49 mins ago














$begingroup$
I checked your result up to $n=533$ (for $n geq 534$, I have problems. Would you be interested by a huge table of $I(k)$ (I was able to generate it up to $k=540$). This is a very interesting problem.
$endgroup$
– Claude Leibovici
30 mins ago






$begingroup$
I checked your result up to $n=533$ (for $n geq 534$, I have problems. Would you be interested by a huge table of $I(k)$ (I was able to generate it up to $k=540$). This is a very interesting problem.
$endgroup$
– Claude Leibovici
30 mins ago














$begingroup$
@ClaudeLeibovici: Thanks for checking, Claude! However, that table would be too huge for MSE. :)
$endgroup$
– Tito Piezas III
27 mins ago




$begingroup$
@ClaudeLeibovici: Thanks for checking, Claude! However, that table would be too huge for MSE. :)
$endgroup$
– Tito Piezas III
27 mins ago










1 Answer
1






active

oldest

votes


















10












$begingroup$

The answer is yes. Sketch of solution:
$$
I(k) = int_0^k sum_{ple x} sum_{qle k-x} 1 ,dx = sum_p sum_{qle k-p} int_p^{k-q} dx = sum_p sum_{qle k-p} (k-(p+q)) = sum_{mle k} r(m)(k-m),
$$

where $r(m)$ is the number of ways of writing $m$ as the sum of two primes. Then
$$
I(6n+6)-2I(6n+5)+I(6n+4) = sum_{mle 6n+4} r(m)big( (6n+6-m)-2(6n+5-m)+(6m+4-m) big) + r(6n+5) = \0 + r(6n+5);
$$

and $r(6n+5)=0$ for every $nge1$, since the only way the odd integer $6n+5$ can be the sum of two primes is $6n+5=2+(6n+3)$, but $6n+3=3(2n+1)$ is always composite when $nge1$.



The same argument gives $I(6n+2)-2I(6n+1)+I(6n) = r(6n+1)$, which is $2$ if $6n-1$ is prime and $0$ otherwise; this is why (as observed by John Omielan) it equals $2$ for $1le nle 5$ but $0$ for $n=6$.






share|cite|improve this answer











$endgroup$









  • 1




    $begingroup$
    MSE never ceases to amaze me how fast some people can figure out the answer.
    $endgroup$
    – Tito Piezas III
    15 mins ago










  • $begingroup$
    Greg, do you know how to address Ultradark's question regarding when $I(k)$ is prime?
    $endgroup$
    – Tito Piezas III
    6 mins ago










  • $begingroup$
    This really surprises me since I thought the equation will be eventually false...
    $endgroup$
    – Seewoo Lee
    33 secs ago











Your Answer





StackExchange.ifUsing("editor", function () {
return StackExchange.using("mathjaxEditing", function () {
StackExchange.MarkdownEditor.creationCallbacks.add(function (editor, postfix) {
StackExchange.mathjaxEditing.prepareWmdForMathJax(editor, postfix, [["$", "$"], ["\\(","\\)"]]);
});
});
}, "mathjax-editing");

StackExchange.ready(function() {
var channelOptions = {
tags: "".split(" "),
id: "69"
};
initTagRenderer("".split(" "), "".split(" "), channelOptions);

StackExchange.using("externalEditor", function() {
// Have to fire editor after snippets, if snippets enabled
if (StackExchange.settings.snippets.snippetsEnabled) {
StackExchange.using("snippets", function() {
createEditor();
});
}
else {
createEditor();
}
});

function createEditor() {
StackExchange.prepareEditor({
heartbeatType: 'answer',
autoActivateHeartbeat: false,
convertImagesToLinks: true,
noModals: true,
showLowRepImageUploadWarning: true,
reputationToPostImages: 10,
bindNavPrevention: true,
postfix: "",
imageUploader: {
brandingHtml: "Powered by u003ca class="icon-imgur-white" href="https://imgur.com/"u003eu003c/au003e",
contentPolicyHtml: "User contributions licensed under u003ca href="https://creativecommons.org/licenses/by-sa/3.0/"u003ecc by-sa 3.0 with attribution requiredu003c/au003e u003ca href="https://stackoverflow.com/legal/content-policy"u003e(content policy)u003c/au003e",
allowUrls: true
},
noCode: true, onDemand: true,
discardSelector: ".discard-answer"
,immediatelyShowMarkdownHelp:true
});


}
});














draft saved

draft discarded


















StackExchange.ready(
function () {
StackExchange.openid.initPostLogin('.new-post-login', 'https%3a%2f%2fmath.stackexchange.com%2fquestions%2f3128367%2fa-curious-equality-of-integrals-involving-the-prime-counting-function%23new-answer', 'question_page');
}
);

Post as a guest















Required, but never shown

























1 Answer
1






active

oldest

votes








1 Answer
1






active

oldest

votes









active

oldest

votes






active

oldest

votes









10












$begingroup$

The answer is yes. Sketch of solution:
$$
I(k) = int_0^k sum_{ple x} sum_{qle k-x} 1 ,dx = sum_p sum_{qle k-p} int_p^{k-q} dx = sum_p sum_{qle k-p} (k-(p+q)) = sum_{mle k} r(m)(k-m),
$$

where $r(m)$ is the number of ways of writing $m$ as the sum of two primes. Then
$$
I(6n+6)-2I(6n+5)+I(6n+4) = sum_{mle 6n+4} r(m)big( (6n+6-m)-2(6n+5-m)+(6m+4-m) big) + r(6n+5) = \0 + r(6n+5);
$$

and $r(6n+5)=0$ for every $nge1$, since the only way the odd integer $6n+5$ can be the sum of two primes is $6n+5=2+(6n+3)$, but $6n+3=3(2n+1)$ is always composite when $nge1$.



The same argument gives $I(6n+2)-2I(6n+1)+I(6n) = r(6n+1)$, which is $2$ if $6n-1$ is prime and $0$ otherwise; this is why (as observed by John Omielan) it equals $2$ for $1le nle 5$ but $0$ for $n=6$.






share|cite|improve this answer











$endgroup$









  • 1




    $begingroup$
    MSE never ceases to amaze me how fast some people can figure out the answer.
    $endgroup$
    – Tito Piezas III
    15 mins ago










  • $begingroup$
    Greg, do you know how to address Ultradark's question regarding when $I(k)$ is prime?
    $endgroup$
    – Tito Piezas III
    6 mins ago










  • $begingroup$
    This really surprises me since I thought the equation will be eventually false...
    $endgroup$
    – Seewoo Lee
    33 secs ago
















10












$begingroup$

The answer is yes. Sketch of solution:
$$
I(k) = int_0^k sum_{ple x} sum_{qle k-x} 1 ,dx = sum_p sum_{qle k-p} int_p^{k-q} dx = sum_p sum_{qle k-p} (k-(p+q)) = sum_{mle k} r(m)(k-m),
$$

where $r(m)$ is the number of ways of writing $m$ as the sum of two primes. Then
$$
I(6n+6)-2I(6n+5)+I(6n+4) = sum_{mle 6n+4} r(m)big( (6n+6-m)-2(6n+5-m)+(6m+4-m) big) + r(6n+5) = \0 + r(6n+5);
$$

and $r(6n+5)=0$ for every $nge1$, since the only way the odd integer $6n+5$ can be the sum of two primes is $6n+5=2+(6n+3)$, but $6n+3=3(2n+1)$ is always composite when $nge1$.



The same argument gives $I(6n+2)-2I(6n+1)+I(6n) = r(6n+1)$, which is $2$ if $6n-1$ is prime and $0$ otherwise; this is why (as observed by John Omielan) it equals $2$ for $1le nle 5$ but $0$ for $n=6$.






share|cite|improve this answer











$endgroup$









  • 1




    $begingroup$
    MSE never ceases to amaze me how fast some people can figure out the answer.
    $endgroup$
    – Tito Piezas III
    15 mins ago










  • $begingroup$
    Greg, do you know how to address Ultradark's question regarding when $I(k)$ is prime?
    $endgroup$
    – Tito Piezas III
    6 mins ago










  • $begingroup$
    This really surprises me since I thought the equation will be eventually false...
    $endgroup$
    – Seewoo Lee
    33 secs ago














10












10








10





$begingroup$

The answer is yes. Sketch of solution:
$$
I(k) = int_0^k sum_{ple x} sum_{qle k-x} 1 ,dx = sum_p sum_{qle k-p} int_p^{k-q} dx = sum_p sum_{qle k-p} (k-(p+q)) = sum_{mle k} r(m)(k-m),
$$

where $r(m)$ is the number of ways of writing $m$ as the sum of two primes. Then
$$
I(6n+6)-2I(6n+5)+I(6n+4) = sum_{mle 6n+4} r(m)big( (6n+6-m)-2(6n+5-m)+(6m+4-m) big) + r(6n+5) = \0 + r(6n+5);
$$

and $r(6n+5)=0$ for every $nge1$, since the only way the odd integer $6n+5$ can be the sum of two primes is $6n+5=2+(6n+3)$, but $6n+3=3(2n+1)$ is always composite when $nge1$.



The same argument gives $I(6n+2)-2I(6n+1)+I(6n) = r(6n+1)$, which is $2$ if $6n-1$ is prime and $0$ otherwise; this is why (as observed by John Omielan) it equals $2$ for $1le nle 5$ but $0$ for $n=6$.






share|cite|improve this answer











$endgroup$



The answer is yes. Sketch of solution:
$$
I(k) = int_0^k sum_{ple x} sum_{qle k-x} 1 ,dx = sum_p sum_{qle k-p} int_p^{k-q} dx = sum_p sum_{qle k-p} (k-(p+q)) = sum_{mle k} r(m)(k-m),
$$

where $r(m)$ is the number of ways of writing $m$ as the sum of two primes. Then
$$
I(6n+6)-2I(6n+5)+I(6n+4) = sum_{mle 6n+4} r(m)big( (6n+6-m)-2(6n+5-m)+(6m+4-m) big) + r(6n+5) = \0 + r(6n+5);
$$

and $r(6n+5)=0$ for every $nge1$, since the only way the odd integer $6n+5$ can be the sum of two primes is $6n+5=2+(6n+3)$, but $6n+3=3(2n+1)$ is always composite when $nge1$.



The same argument gives $I(6n+2)-2I(6n+1)+I(6n) = r(6n+1)$, which is $2$ if $6n-1$ is prime and $0$ otherwise; this is why (as observed by John Omielan) it equals $2$ for $1le nle 5$ but $0$ for $n=6$.







share|cite|improve this answer














share|cite|improve this answer



share|cite|improve this answer








edited 16 mins ago









Tito Piezas III

27.4k366174




27.4k366174










answered 29 mins ago









Greg MartinGreg Martin

35.5k23263




35.5k23263








  • 1




    $begingroup$
    MSE never ceases to amaze me how fast some people can figure out the answer.
    $endgroup$
    – Tito Piezas III
    15 mins ago










  • $begingroup$
    Greg, do you know how to address Ultradark's question regarding when $I(k)$ is prime?
    $endgroup$
    – Tito Piezas III
    6 mins ago










  • $begingroup$
    This really surprises me since I thought the equation will be eventually false...
    $endgroup$
    – Seewoo Lee
    33 secs ago














  • 1




    $begingroup$
    MSE never ceases to amaze me how fast some people can figure out the answer.
    $endgroup$
    – Tito Piezas III
    15 mins ago










  • $begingroup$
    Greg, do you know how to address Ultradark's question regarding when $I(k)$ is prime?
    $endgroup$
    – Tito Piezas III
    6 mins ago










  • $begingroup$
    This really surprises me since I thought the equation will be eventually false...
    $endgroup$
    – Seewoo Lee
    33 secs ago








1




1




$begingroup$
MSE never ceases to amaze me how fast some people can figure out the answer.
$endgroup$
– Tito Piezas III
15 mins ago




$begingroup$
MSE never ceases to amaze me how fast some people can figure out the answer.
$endgroup$
– Tito Piezas III
15 mins ago












$begingroup$
Greg, do you know how to address Ultradark's question regarding when $I(k)$ is prime?
$endgroup$
– Tito Piezas III
6 mins ago




$begingroup$
Greg, do you know how to address Ultradark's question regarding when $I(k)$ is prime?
$endgroup$
– Tito Piezas III
6 mins ago












$begingroup$
This really surprises me since I thought the equation will be eventually false...
$endgroup$
– Seewoo Lee
33 secs ago




$begingroup$
This really surprises me since I thought the equation will be eventually false...
$endgroup$
– Seewoo Lee
33 secs ago


















draft saved

draft discarded




















































Thanks for contributing an answer to Mathematics Stack Exchange!


  • Please be sure to answer the question. Provide details and share your research!

But avoid



  • Asking for help, clarification, or responding to other answers.

  • Making statements based on opinion; back them up with references or personal experience.


Use MathJax to format equations. MathJax reference.


To learn more, see our tips on writing great answers.




draft saved


draft discarded














StackExchange.ready(
function () {
StackExchange.openid.initPostLogin('.new-post-login', 'https%3a%2f%2fmath.stackexchange.com%2fquestions%2f3128367%2fa-curious-equality-of-integrals-involving-the-prime-counting-function%23new-answer', 'question_page');
}
);

Post as a guest















Required, but never shown





















































Required, but never shown














Required, but never shown












Required, but never shown







Required, but never shown

































Required, but never shown














Required, but never shown












Required, but never shown







Required, but never shown







Popular posts from this blog

is 'sed' thread safeWhat should someone know about using Python scripts in the shell?Nexenta bash script uses...

How do i solve the “ No module named 'mlxtend' ” issue on Jupyter?

Pilgersdorf Inhaltsverzeichnis Geografie | Geschichte | Bevölkerungsentwicklung | Politik | Kultur...